Anuncio

Colapsar
No hay ningún anuncio todavía.

Transformada de Laplace

Colapsar
X
 
  • Filtro
  • Hora
  • Mostrar
Borrar todo
nuevos mensajes

  • Otras carreras Transformada de Laplace

    Determine F(s) para la f(t):



    Alguna ayuda?

    Lo unico que se es que tengo que partir de :



    como hago con la funcion escalon que aparece ahi?

    Alguien sabe resolverla?

    gracias
    Última edición por LauraLopez; 24/06/2013, 23:03:33.

  • #2
    Re: Transformada de Laplace

    Hola:

    No se del tema, pero usando la logica podemos decir que si:



    entonces podemos decir que:

    [Error LaTeX: Compilación LaTeX fallida]

    por lo cual:



    dentro de todo el intervalo de integracion la funcion escalon es igual a 1, por lo cual:



    [Error LaTeX: Compilación LaTeX fallida]



    Para s<1 resulta que 1-s>0 por lo cual el 1º termino de la transformada diverge, por lo cual la transformada no existe para s<1.

    Ahora para s>1 tenemos que 1-s<0 y el 1º termino de la transformada se anula y la transformada converge y resulta que:



    Espero no estar muy desacertado, y que se entienda.

    Suerte
    No tengo miedo !!! - Marge Simpson
    Entonces no estas prestando atención - Abe Simpson

    Comentario


    • #3
      Re: Transformada de Laplace

      si fue un error de tipeo es negativo el exponente, y perdon pero el enunciado tambien cometi un error de tipeo es :



      Si no te entendi mal entonces como la funcion escalon vale 1 en ese intervalo simplemente seria como eliminarla no? o sea debo hacer

      seria eso no?

      luego si es asi intento resolverlo y creo que seria asi...



      esta bien?

      Gracias
      Última edición por LauraLopez; 25/06/2013, 01:35:00.

      Comentario


      • #4
        Re: Transformada de Laplace

        Hola:

        No revise el resultado, pero el procedimiento parece estar bien (las integrales a resolver son sencillas). Si estas están bien lo único que falta expresar es para que valores de s es valida (converge) la transformada.

        Suerte
        No tengo miedo !!! - Marge Simpson
        Entonces no estas prestando atención - Abe Simpson

        Comentario


        • #5
          Re: Transformada de Laplace

          y como se sabe para que valores converge?

          Comentario


          • #6
            Re: Transformada de Laplace

            Hola:

            Por ejemplo:

            [Error LaTeX: Compilación LaTeX fallida]



            Para 9+s<0 el 1º termino de la ultima expresión se hace infinito, y la transformada no converge (no existe) para estos valores de s.

            Sin embargo para 9+s>0 este miembro se hace cero y la transformada resulta:



            Tenes que hacer lo mismo para todas las integrales, encontrar para que valores de s existe la transformada de cada integral.

            La intersección de los conjuntos de valores de s donde es valida la transformada de cada función (cada integral), es el conjunto de valores de s donde es valida la transformada de la suma de funciones.

            Suerte
            Última edición por Breogan; 25/06/2013, 02:39:21.
            No tengo miedo !!! - Marge Simpson
            Entonces no estas prestando atención - Abe Simpson

            Comentario


            • #7
              Re: Transformada de Laplace

              mmmm tengo un par de dudas ,o sea la solucion siempre es dar una F(s) y decir para que intervalos es valida? los intervalos en lo que es valida son los intervalos donde no valen cero los denominadores? no estoy segura si te entendi....


              Resolviendo el ejercicio llego a ( no estoy segura del resultado despues si alguien sabe hacer Laplace espero me pueda ayudar a corroborarlo )



              entonces ahora debo decir para que intervalos esto es valido? creo que es valido para todo s distinto de 9 y distinto de +1 y de -1, no es asi?

              Comentario


              • #8
                Re: Transformada de Laplace

                Hola:

                Voy a tratar de resolver, según lo poco que se del tema, paso a paso el problema a ver si despejas algunas dudas.



                La transformada es:



                dentro de todo el intervalo de integración resulta que , por lo cual la integral anterior queda:



                haciendo la distributiva dentro de la integral:



                agrupando:



                resolviendo las integrales:



                aplicando la regla de Barrow:



                eliminando parentesis:



                analizando los términos de la ultima ecuación se ve que:

                1º Termino



                2º Termino



                3º Termino




                4º Termino




                5º Termino




                6º Termino




                Por lo cual la transformada de Laplace queda:



                y este es valido cuando , que es la intersección de los rangos de validez hallados al analizar los valores de cada termino en la transformación.
                Es decir que s>1 garantiza que los valores hallados para los seis terminos son validos al mismo tiempo.

                Espero que se entienda

                Suerte
                No tengo miedo !!! - Marge Simpson
                Entonces no estas prestando atención - Abe Simpson

                Comentario


                • #9
                  Re: Transformada de Laplace

                  excelente explicacion Breogan! gracias por el detalle me fue muy util para ser el primer ejercicio de Laplace. Una sola pequeña duda, cuando das las restriccion de que s sea real porque es? no podria ser s complejo? donde aclaro que su parte real deberia ser mayor que 1?

                  Gracias

                  Comentario


                  • #10
                    Re: Transformada de Laplace

                    Hola:

                    Para mi tambien es el 1º ejercicio de este tipo, por lo menos que me acuerde.

                    Escrito por LauraLopez Ver mensaje
                    . Una sola pequeña duda, cuando das las restriccion de que s sea real porque es?
                    Cuando se define la transformada de Laplace, se dice que s debe pertenecer a los números reales, por esto ya queda justificada la restricción a los reales.

                    Escrito por LauraLopez Ver mensaje
                    no podria ser s complejo?
                    Realmente esta fuera de mi alcance responder esto, tengo una ligera idea, pero para no complicarla me quedaría con la definición: s pertenece a los reales

                    Escrito por LauraLopez Ver mensaje
                    donde aclaro que su parte real deberia ser mayor que 1?
                    Cuando escribís la ecuación de la solución también tenes que declarar para que valores de s es valida. Aclaración: no es la parte real de nada, es s>1

                    NOTA: en este caso la transformada de Laplace no tiene polos (puntos donde la transformada se hace infinito, p.e. s=-9) dentro del dominio de validez en s; creo que hay casos en que la transformada puede tener polos dentro de su dominio de validez y estos son motivo de un análisis ulterior, si es así ya aparecerá en tu libro mas adelante y no te preocupes; si no es así disculpas .

                    Suerte
                    No tengo miedo !!! - Marge Simpson
                    Entonces no estas prestando atención - Abe Simpson

                    Comentario

                    Contenido relacionado

                    Colapsar

                    Trabajando...
                    X